Tensor of Inertia for Half Disk: Calc Angular Momentum

  • Thread starter Thread starter springo
  • Start date Start date
  • Tags Tags
    Inertia Tensor
Click For Summary
The discussion centers on calculating the tensor of inertia for a half disk with mass M and determining the angular momentum along a specified axis. The initial moment of inertia for a full disk is provided, and an attempt to derive the tensor for the half disk is made using Steiner's theorem. The proposed tensor is adjusted, but the calculations lead to confusion regarding the angular momentum, as the results suggest no variation at constant rotation speed. The user seeks clarification on the correct application of the parallel axis theorem to the inertia tensor. Accurate calculations are essential for deriving the correct angular momentum.
springo
Messages
125
Reaction score
0

Homework Statement


Find the tensor of inertia for a half disk with mass M and then use that to get the angular momentum along the axis in the figure.
http://img138.imageshack.us/img138/5312/problemr.png


Homework Equations


Moment of inertia for the whole disk (with mass 2M)
\begin{bmatrix}<br /> \frac{MR^{2}}{2} &amp; 0 &amp; 0 \\ <br /> 0 &amp; \frac{MR^{2}}{2} &amp; 0 \\ <br /> 0 &amp; 0 &amp; MR^{2}<br /> \end{bmatrix}

The Attempt at a Solution


For the tensor:
\begin{bmatrix}<br /> \frac{MR^{2}}{2} &amp; 0 &amp; 0 \\ <br /> 0 &amp; \frac{MR^{2}}{4} &amp; 0 \\ <br /> 0 &amp; 0 &amp; \frac{MR^{2}}{2}<br /> \end{bmatrix}
And the moment of inertia... I think I should apply Steiner's theorem, but I'm not quite sure how to apply it on a tensor.

Thanks for your help.
 
Last edited by a moderator:
Physics news on Phys.org
See:
http://en.wikipedia.org/wiki/Parallel_axis_theorem"

You take the displacement vector \mathbf{a}

(written as a column vector) and form the matrix \mathbf{a}\mathbf{a}^\top

The new moment of inertia tensor is then:
\mathbf{I}&#039;=\mathbf{I} + M(|a|^2\mathbf{1} -\mathbf{a}\mathbf{a}^\top)

[edited above, forgot the mass!]
 
Last edited by a moderator:
Thanks.
OK, so assuming my "attempt at a solution" tensor was right, I get to the following tensor:

<br /> \begin{bmatrix}<br /> \frac{MR^{2}}{2} &amp; 0 &amp; 0 \\ <br /> 0 &amp; \frac{MR^{2}}{4} &amp; 0 \\ <br /> 0 &amp; 0 &amp; \frac{MR^{2}}{2}<br /> \end{bmatrix}<br /> +\begin{bmatrix}<br /> 0 &amp; 0 &amp; 0 \\ <br /> 0 &amp; MR^{2} &amp; 0 \\ <br /> 0 &amp; 0 &amp; MR^{2}<br /> \end{bmatrix}<br /> =\begin{bmatrix}<br /> \frac{MR^{2}}{2} &amp; 0 &amp; 0 \\ <br /> 0 &amp; \frac{5MR^{2}}{4} &amp; 0 \\ <br /> 0 &amp; 0 &amp; \frac{3MR^{2}}{2}<br /> \end{bmatrix}<br />

I think this can't be right because then when I try to find variation in the angular momentum (at constant rotation speed), I get 0.
 
Last edited:
Question: A clock's minute hand has length 4 and its hour hand has length 3. What is the distance between the tips at the moment when it is increasing most rapidly?(Putnam Exam Question) Answer: Making assumption that both the hands moves at constant angular velocities, the answer is ## \sqrt{7} .## But don't you think this assumption is somewhat doubtful and wrong?

Similar threads

Replies
11
Views
3K
  • · Replies 9 ·
Replies
9
Views
899
Replies
3
Views
1K
Replies
335
Views
15K
  • · Replies 12 ·
Replies
12
Views
2K
  • · Replies 6 ·
Replies
6
Views
2K
  • · Replies 4 ·
Replies
4
Views
1K
Replies
1
Views
2K
  • · Replies 3 ·
Replies
3
Views
2K
  • · Replies 2 ·
Replies
2
Views
1K